Trying to find the Laplace transform of \(\displaystyle{\frac{{{\cos{{t}}}}}{{{t}}}}\) It

Mary Bates

Mary Bates

Answered question

2022-04-08

Trying to find the Laplace transform of
costt
It comes out as infinity, but that doesn't make any sense.
Does this mean that this function doesn't have a Laplace transform or is something wrong here?

Answer & Explanation

Jax Burns

Jax Burns

Beginner2022-04-09Added 13 answers

Well, we are trying to find (solving a more general problem):
Lt[cos(nt)t](s)=0cos(nt)texp(st)dx
Using the 'frequency-domain integration' property of the Laplace transform, we can write:
Lt[cos(nt)t](s)=sLt[cos(nt)](σ)dσ
Using the table of selected Laplace transforms, we find:
Lt[cos(nt)](σ)=σσ2+n2
So, we get:
Lt[cos(nt)t](s)=sσσ2+n2dσ
Let's substitute u=σ2+n2 , so we get:
Lt[cos(nt)t](s)=12limϵs2+n2ϵ2+n21udu=12limϵ[ln|u|]s2+n2ϵ2+n2=
12limϵ(ln|ϵ2+n2|ln|s2+n2|)=12limϵ|ϵ2+n2s2+n2|

Do you have a similar question?

Recalculate according to your conditions!

New Questions in Differential Equations

Ask your question.
Get an expert answer.

Let our experts help you. Answer in as fast as 15 minutes.

Didn't find what you were looking for?